LSAT and Law School Admissions Forum

Get expert LSAT preparation and law school admissions advice from PowerScore Test Preparation.

User avatar
 Dave Killoran
PowerScore Staff
  • PowerScore Staff
  • Posts: 5853
  • Joined: Mar 25, 2011
|
#79283
Complete Question Explanation
(The complete setup for this game can be found here: lsat/viewtopic.php?t=17034)

The correct answer choice is (B).

The question stem requires that you find a solution that has no “vertical” connector. Only Solution #5 does not contain a vertical connector. Answer choices (A), (C), (D), and (E) all describe connections contained within Solution #5, and so answer choice (B) is not necessarily true (in fact, (B) cannot be true), and is therefore the correct answer.
 AJH
  • Posts: 15
  • Joined: Nov 20, 2017
|
#43606
The diagram I had for this question is M to J, J to N, N to O, O to K and K to L. I don't see how N & O can not not be connected. The other alternative would be for L to connect O, which violates the rule in this question that they can't connect directly opposite the row. Otherwise, you would have to cross a segment to get O looped in which violates one of the initial rules. I think I am missing a template, but was able to answer all the other questions with ease with this one template. Please help me see the step I am missing! Thanks!
 James Finch
PowerScore Staff
  • PowerScore Staff
  • Posts: 943
  • Joined: Sep 06, 2017
|
#43704
Hi AJH,

So your diagram immediately breaks the rules by connecting M-J across the rows. Instead, M may only connect with N, thus taking up both of N's possible connections (it must always connect to J), and not allowing it to connect to O at all. Which leaves O looking stranded, and also the key to this question. O must connect to both J and K. The correct diagram ends up being:

M-N
N-J
J-O
O-K
K-L

Hope this clears things up!
 Goshen574!
  • Posts: 1
  • Joined: Dec 07, 2019
|
#72465
Originally, I misunderstood the question. Upon further evaluation, I realize that it is a must be false question, the remainder could be true.

With that said, I still cannot understand why the answer to question 24 is Choice B, and not seeing? Why is CA could be true in this case? I understand that answer choices A, D, and E are could be trues and therefor are “wrong” but why is answer C wrong?

In the template you gave us and the question clearly shows that L cannot have another person attached to it...so what am I getting wrong?
 Adam Tyson
PowerScore Staff
  • PowerScore Staff
  • Posts: 5153
  • Joined: Apr 14, 2011
|
#72467
Thanks for the question, Goshen574! First, let me clear up the question type for you. This is not a Must Be False question (aka a Cannot Be True). Rather, it is a Must Be True-EXCEPT question, which means that the four wrong answers MUST be true, while the correct answer is Not Necessarily True (aka Could Be False). So, we are looking for the one answer that doesn't have to be true.

Check out James' breakdown of the local diagram for this question, and that should show you that answer C MUST be true. L has to be connected to exactly one other chalet, which is K as per the original rules. It cannot be connected to O, because the local question forbids any chalets directly across from each other from being connected. So L is definitely directly connected to exactly one other chalet.

Get the most out of your LSAT Prep Plus subscription.

Analyze and track your performance with our Testing and Analytics Package.